Középiskolai Matematikai és Fizikai Lapok
Informatika rovattal
Kiadja a MATFUND Alapítvány
Már regisztráltál?
Új vendég vagy?

Fórum: Nehezebb matematikai problémák

  [1]    [2]    [3]    [4]    [5]    [6]    [7]    [8]    [9]    [10]    [11]    [12]    [13]    [14]    [15]    [16]    [17]    [18]    [19]    [20]    [21]    [22]    [23]    [24]    [25]    [26]    [27]    [28]    [29]    [30]    [31]    [32]    [33]    [34]  

Szeretnél hozzászólni? Jelentkezz be.
[759] marcius82014-01-24 12:46:40

Még egy újabb kérdés jutott az eszembe, de ez már geometria. Adott három egyforma sugarú és végtelen hosszú henger. A hengerek forgástengelyei páronként egymásra merőlegesek és egy pontban metszik egymást. Tekintsük azt a testet, amely egyenlő a három henger közös részével (metszetével). Mekkora ennek a testnek a felszíne és a térfogata?

[758] marcius82014-01-24 12:42:16

Egy újabb kérdés jutott az eszembe.

A.) Aprajafalván a toronyóra mutatói (kismutató és nagymutató) cm-ben mérve egész számok. 3 óra időtájban a két mutató végpontjának távolsága cm-ben mérve szintén egész szám. 2 óra időtájban a két mutató végpontjának távolsága cm-ben mérve szintén egész szám. Mekkora lehet a két mutató hossza?

B.) Aprajafalván a toronyóra mutatói (kismutató és nagymutató) cm-ben mérve egész számok. 3 óra időtájban a két mutató végpontjának távolsága cm-ben mérve szintén egész szám. 4 óra időtájban a két mutató végpontjának távolsága cm-ben mérve szintén egész szám. Mekkora lehet a két mutató hossza?

Ráadásként:

C.) Aprajafalván a toronyóra mutatói (kismutató és nagymutató) cm-ben mérve egész számok. 2 óra időtájban a két mutató végpontjának távolsága cm-ben mérve szintén egész szám. 4 óra időtájban a két mutató végpontjának távolsága cm-ben mérve szintén egész szám. Mekkora lehet a két mutató hossza?

Ha valakinek van ezeknek a kérdéseknek a megoldásához ötlete, azt előre is köszönöm. Bertalan Zoltán.

[757] aaaa2014-01-23 20:36:23

Legyen E(X) a keresett várható érték, és keressük a k fej egymás utáni dobásának várható értékét. Ezt megfeltételezzük (feltételes várható érték tulajdonságai miatt ez menni fog), mégpedig az alapján, hogy az első írásig hány fejet dobunk. Ekkor teljesül


E(X)=kp^k+\sum_{i=0}^{k-1} p^i (1-p) (E(X)+i+1)=\frac{(1-p^k)(1+(1-p)E(x))}{1-p}

(Az összeget deriválást alkalmazó trükkel lehet könnyen számolni) Ezt kell megoldani E(X)-re, ekkor kapjuk, hogy

E(X)=\frac{1-p^k}{p^k(1-p)}=\frac{1+p+\dots+p^{k-1}}{p^k}

Előzmény: [756] Liklihood, 2014-01-16 13:41:28
[756] Liklihood2014-01-16 13:41:28

A következő feladat megoldásában kérném a segítségeteket. Legyen egy cinkelt pénzérménk, melyre P(fej)=p=1-P(írás), ahol 0<p<1. Várhatóan hanyadik dobásra következik be, hogy 5 fejet dobunk egymás után?

Válaszaitokat előre is köszönöm. :)

[755] marcius82014-01-07 10:14:14

Köszönöm a hozzászólást. Igazából csak addig jutottam én is, hogy hogyan nézhet ki egy ilyen gráf, és arra jutottam, hogy egy ilyen gráf diszjunkt körökből áll. Tisztelettel: Bertalan Zoltán.

Előzmény: [753] aaaa, 2013-12-23 01:12:02
[754] aaaa2013-12-23 02:19:16

Némi elírás, a megadott képlet minden gráfra számol, és l-et 3-től kell indítani.

Javítás páros gráfra

Párosra csak a páros tagokat kell összegezni, ezeket is csak 2-től kezdve, így a következőképpen módosulnak a dolgaink:

T(l)=1+\frac{x^{l}}{2l}e^{x^{l}(2l)^{-1}}

G(x)=\prod_{i=2}^\infty  \left(1+\frac{x^{2i}}{4i}e^{x^{2i}(4i)^{-1}}\right)

És G(x)-ben xn együtthatója adja meg a sorrendek számát n!-al osztva.

Előzmény: [753] aaaa, 2013-12-23 01:12:02
[753] aaaa2013-12-23 01:12:02

Egy előzőhöz hasonló gondolatmenettel valahogy így kellene:

Most irányított gráfra csináljuk: Ha meg meg vannak különböztetve a csúcsok, akkor ugye ezeknek n! sorrendje van. Ezt ugye daraboljuk egy partíció szerint. Mit számolunk többször? Hát, ha ugyanazok a csúcsok más sorrendben, de ugyanabban a ciklikus permutációnak megfelelő sorrendben vannak, illetve ha ugynakkora méretű halmazaink vannak, csak más sorrendben. Most generátorfüggvényt csinálunk, \frac{a_n}{n!}-ra:

Először csak azt vesszük bele a játékba, hogy k darab ugyanakkora halmazt (k-1)! alkalommal számoltunk, vagyis a hatványsora valahogy így nézzen ki egy tényező:

U(l)=1+\sum_{k=1}^\infty \frac{x^{lk}}{(k-1)!}

Viszont ezeknek lk darab egymástól független, ugyanolyan gráfot eredményező sorrendje van, tehát:

T(l)=1+\sum_{k=1}^\infty \frac{x^{lk}}{(k-1)!l^k}=1+\frac{x^l}{l}e^{x^{l}l^{-1}}

A keresett generátorfüggvény tehát úgy áll elő, hogy a

g(l)=\prod_{l=2}^{\infty}1+\frac{x^l}{l}e^{x^{l}l^{-1}}

Szorzat xn-hez tartozó együtthatóját szorozzuk n!-al, és elvileg készen kellene lennünk. Ha meg irányításfüggetlen, akkor ennek pontosan a fele jó sorrend, ekkor

G(l)=\prod_{l=2}^{\infty}\left(1+\frac{x^l}{2l}e^{x^{l}(2l)^{-1}}\right)

Előzmény: [752] aaaa, 2013-12-23 00:24:11
[752] aaaa2013-12-23 00:24:11

Kb. 2 percet gondolkozva rajta a következőre jutottam: Mivel a gráf páros, minden kör 2k hosszú, ahol k\geq2, és ez elég is a párossághoz. Címkézetlen csúcsokon vagyunk, így lényegében n pont partíciónak a száma a kérdés, ahol minden egyes részhalmaz legalább 2 elemet tartalmaz. Ennek a generátorfüggvénye meg:

g(x)=\prod_{i=2}^\infty\frac{1}{1-x^i}

Aminek a hatványsorának xi-hez tartozó együtthatója épp a 2i-re a nem izomorf fák számát. Ennek első néhány tagja:

g(x)\approx1+x2+x3+2x4+2x5+4x6+4x7+7x8+8x9+12x10+14x11+21x12+24x13+34x14+41x15+55x16+66x17+88x18+105x19+137x20+O[x]21

Itt megtalálod a sorozat néhány következő tagját. Szerintem ez megadja az izomorfia erejéig a jó gráfok számát, csak ki kell fejteni. Azt nem hinném, hogy ennél sokkal explicitebb képlet létezik, mivel azt írja, hogy P(n+1)-P(n) sorozat ez lesz, ahol P(n) a partíciók száma. Kicsit fáradt vagyok már, szóval lehet, hogy valamit elnéztem.

Előzmény: [750] marcius8, 2013-12-20 10:04:05
[751] csábos2013-12-22 23:35:36

Nem értem a kérdést. Adott a páros gráf? Akkor mik az élei? Ha nem adott, akkor adott két db n-elemű halmaz, és ezen hány olyan páros gráf van, amely diszjunkt körök uniója? Ha a gráf adott, meg kell-e őrizni a paritását?

Előzmény: [750] marcius8, 2013-12-20 10:04:05
[750] marcius82013-12-20 10:04:05

Tekintsünk egy páros gráfot, melynek "2n" csúcsa van. Hányféleképpen lehet ebbe a páros gráfba berajzolni az éleket, úgy hogy minden csúcs foka 2 legyen? Kicsit nehezebb kérdés: ezen lehetőségek között mennyi egymással nem izomorf gráf van? Ha valakinek van ötlete ezzel kapcsolatban, kérem írja le. Előre is köszönettel: Bertalan Zoltán.

[749] marcius82013-12-20 10:00:36

Tudjuk, hogy ha egy tórusz középkörének sugara "R", meridiánkörének sugara "r", ahol "r" kisebb "R" egyenlőtlenség teljesül (klasszikus úszógumi), akkor a tórusz felszíne 4*pi*pi*R*r és a tórusz térfogata 2*pi*pi*R*r*r. Azt is tudjuk, hogy a pozitív egész számok négyzetének reciprokösszege pi*pi/6. Vajon ez utóbbi állítást nem lehetséges bebizonyítani a tórusz felszínképletének vagy térfogatképletének felhasználásával, tekintettel arra, hogy a "pi*pi" mennyiség itt is és ott is megjelenik? Ha valakinek van ötlete, és megírja, nagyon megköszönöm. Tisztelettel: Bertalan Zoltán.

[748] Cogito2013-12-13 14:48:22

Magamat is javítom.

:-)

A fekete hátterű egyenlőtlenség helyére ezeket kell írni:

a\geb\gec, ha a+b+c\ge0

illetve:

a\leb\lec, ha a+b+c\le0.

Előzmény: [745] Cogito, 2013-12-09 00:40:38
[747] sakkmath2013-12-11 00:56:20

A feladat szerzője Vasile Cirtoaje. ALGEBRAIC INEQUALITIES - OLD AND NEW METHODS című művének 67. oldalán a feladat első közlése olvasható, a 70 - 73. oldalakon a részletes megoldást élvezhetjük.

Előzmény: [739] w, 2013-11-29 22:31:58
[746] HoA2013-12-09 16:10:51

Mellesleg javaslom, angol szaknyelvet senki se az idézett cikkből próbáljon tanulni.

Előzmény: [745] Cogito, 2013-12-09 00:40:38
[745] Cogito2013-12-09 00:40:38

Egyetértek, a "2. megoldásbeli" azonosság önmagában tényleg bosszantóan kevés.

Az interneten itt olvashatjuk a The Interesting Around Technical Analysis Three Variable Inequalities - Nguyen Duy Tung, Zhou Yuan Zhe.pdf-et, melynek 10-11. oldalán közlik a feladat egy - még mindig nem teljes és sajnos (sajtó)hibás - megoldását. E hibás helyeket negatívba fordítottam. Ide írom a javításukat:

A fekete hátterű részeknél: az egyenlőtlenség helyesen a\geb\gec, a 2-esek helyére pedig 1-esek írandók.

Kiegészítésül egy azonosság, amit itt érdemes ismerni: a3b + b3c + c3a - (ab3 + bc3 + ca3) = (a + b + c)(a - c)(c - b)(b - a).

Előzmény: [741] w, 2013-12-02 20:58:17
[744] csábos2013-12-07 22:24:13

Gyors vagy

Előzmény: [743] w, 2013-12-07 22:19:33
[743] w2013-12-07 22:19:33

Mivelhogy a \sqrt x függvény konkáv, így a rá felírt általad mondott Jensen a következőképp néz ki: \sqrt{\frac{\sum a/b\cdot a^2b^2}{\sum a/b}}\ge \frac{\sum a/b\cdot \sqrt{a^2b^2}}{\sum a/b}, emberibb formában \sqrt{\frac{\sum a^3b}{S}}\ge \frac{\sum a^2}{S}, ahol S=\frac ab+\frac bc+\frac ca, amit tovább átírva adódik S\left(a^3b+b^3c+c^3a\right)\ge (a^2+b^2+c^2)^2. Szóval nem: ez szimplán egy másik egyenlőtlenség.

Előzmény: [742] csábos, 2013-12-07 22:09:50
[742] csábos2013-12-07 22:09:50

Nagyon elnézhetek valamit:

ez nem a Jensen-egyenlőtlenség a \sqrt x \frac{}{} függvényre az a^2b^2\frac{}{}, stb helyekre \frac{a}{b} stb.súlyokkal?

Előzmény: [739] w, 2013-11-29 22:31:58
[741] w2013-12-02 20:58:17

Igen, ez így van, de azzal egyetérthetsz, hogy a "2. megoldásbeli" azonosságot egymagában kicsit szemetebb dolog odavágni. :-) A két megoldás azonban olyan szempontból különbözik egymástól, hogy teljesen eltérő módon lehet rájönni - az 1. megoldás kitalálhatóbb, ha az említett egyenlőtlenségből indulunk ki, a második pedig az egyenlőség-esetből vezeti le a kívánt állítást (ami egyébként önmagában nehéz feladat). A megoldás kialakulásáról ebben a beszélgetésben olvashatunk.

Előzmény: [740] Cogito, 2013-12-02 20:10:08
[740] Cogito2013-12-02 20:10:08

Nézzük, hogyan jutunk 1-ről a 2-re:

Az "1. megoldás"-ban szereplő triviális egyenlőtlenséggel ekvivalens:

\frac12 \Big[\big(x-y)^2+\big(y-z)^2+\big(z-x)^2\Big]\ge 0,

ahová x, y, z értékeit behelyettesítve: \frac12 \Big[\big(a^2+2bc-b^2-ca-ab)^2+\big(b^2+2ca-c^2-ab-bc)^2+\big(c^2+2ab-a^2-bc-ca)^2\Big]\ge 0

adódik, ahol a bal oldalon álló kifejezés éppen A. Az első "megoldás" tehát ekvivalens a másodikkal! Ezért - valójában - egyetlen "megoldást" mutattál be. Azért használom az idézőjeleket, mert hiányérzetem van. Nem mutattad meg, hogy milyen út, milyen meggondolások vezettek az x-re, y-ra és z-re adott helyettesítésekhez.

Előzmény: [739] w, 2013-11-29 22:31:58
[739] w2013-11-29 22:31:58

Állítás: \left(a^2+b^2+c^2\right)^2\ge 3\left(a^3b+b^3c+c^3a\right).

1. megoldás. Hasonlítsuk össze egy triviális egyenlőtlenséggel, azaz hogy (x+y+z)2\ge3(xy+yz+zx) tetszőleges valós x,y,z-re. Ebbe x=a2+bc-ab, y=b2+ca-bc, z=c2+ab-ca értékeket helyettesítve pedig éppen a bizonyítandó adódik!

2. megoldás. Vegyük észre, hogy

A=\left(a^2+b^2+c^2\right)^2-3\left(a^3b+b^3c+c^3a\right)=

=\frac{1}{2}\left(\left( a^{2}-2ab+bc-c^{2}+ca\right)^2+\left(b^{2}-2bc+ca-a^{2}+ab\right)^2+\left( c^{2}-2ca+ab-b^{2}+bc\right)^2\right)\ge0.

Mit szóltok?

Előzmény: [733] w, 2013-10-25 23:07:23
[738] marcius82013-11-19 12:59:29

Köszönöm a példát. Én is valami ilyesmire gondoltam, de nem tudtam ilyen precízen megfogalmazni. Tisztelettel: Bertalan Zoltán

Előzmény: [723] w, 2013-10-08 14:56:22
[737] HoA2013-11-18 15:26:19

A követelmény úgy is megfogalmazható, hogy minden kiválasztott négyzetnek páros, minden nem kiválasztott négyzetnek páratlan számú kiválasztott szomszédja legyen.

Előzmény: [734] kurthyg, 2013-11-17 00:44:52
[736] kurthyg2013-11-17 11:04:40

A kiválasztandó négyzetek sorozata. Számozzuk be a négyzeteket: a bal fölső sarok az 1-es a jobb alsó nxn: tehát balról jobbra és föntről lefelé növekszenek a számok.

Ekkor az út (ha eredetileg minden négyzet fehér volt): 1 3 5 7 9

Ezeket sorban kiválasztva minden négyzet fekete lesz.

Előzmény: [735] Sinobi, 2013-11-17 10:48:31
[735] Sinobi2013-11-17 10:48:31

mi az, hogy út?

Előzmény: [734] kurthyg, 2013-11-17 00:44:52

  [1]    [2]    [3]    [4]    [5]    [6]    [7]    [8]    [9]    [10]    [11]    [12]    [13]    [14]    [15]    [16]    [17]    [18]    [19]    [20]    [21]    [22]    [23]    [24]    [25]    [26]    [27]    [28]    [29]    [30]    [31]    [32]    [33]    [34]